Prove $sum_kmid nmu(k)d(k)=(-1)^omega(n)$ Announcing the arrival of Valued Associate #679: Cesar Manara Planned maintenance scheduled April 17/18, 2019 at 00:00UTC (8:00pm US/Eastern)Showing $sum_dmid n mu(d)tau(n/d)=1$ and $sum_dmid n mu(d)tau(d)=(-1)^r$Möbius function verificationProve $sum_k = 1^n mu(k)left[ frac nk right] = 1$Convolution identity involving the Möbius function $sum_n,d>0 |mu(d)| = 2^omega(n)$Prove that $sum_t vert n d^3(t) = (sum_t vert nd(t))^2$ for all $n in mathbbN$Proof of inequality involving multiplicative function?Bound for the sum of the divisors of a numberProve that $sum_n, d geq 1 = 2^omega(n)$Formula for unique distribution of colored balls into boxesUpper bound for the divisor counting function?How to invert an arithmetic function where Möbius inversion may not apply?

Cold is to Refrigerator as warm is to?

Is drag coefficient lowest at zero angle of attack?

Can a zero nonce be safely used with AES-GCM if the key is random and never used again?

Stars Make Stars

Who can trigger ship-wide alerts in Star Trek?

Stop battery usage [Ubuntu 18]

Is dark matter really a meaningful hypothesis?

How do I automatically answer y in bash script?

If A makes B more likely then B makes A more likely"

Direct Experience of Meditation

Blender game recording at the wrong time

Is there a service that would inform me whenever a new direct route is scheduled from a given airport?

Unexpected result with right shift after bitwise negation

How does the Nova's Burn power work at the 7-9 level?

When is phishing education going too far?

Fishing simulator

Why is there no army of Iron-Mans in the MCU?

Slither Like a Snake

Mortgage adviser recommends a longer term than necessary combined with overpayments

Cauchy Sequence Characterized only By Directly Neighbouring Sequence Members

How do we build a confidence interval for the parameter of the exponential distribution?

What loss function to use when labels are probabilities?

Are my PIs rude or am I just being too sensitive?

What are the performance impacts of 'functional' Rust?



Prove $sum_kmid nmu(k)d(k)=(-1)^omega(n)$



Announcing the arrival of Valued Associate #679: Cesar Manara
Planned maintenance scheduled April 17/18, 2019 at 00:00UTC (8:00pm US/Eastern)Showing $sum_dmid n mu(d)tau(n/d)=1$ and $sum_dmid n mu(d)tau(d)=(-1)^r$Möbius function verificationProve $sum_k = 1^n mu(k)left[ frac nk right] = 1$Convolution identity involving the Möbius function $sum_d |mu(d)| = 2^omega(n)$Prove that $sum_t vert n d^3(t) = (sum_t vert nd(t))^2$ for all $n in mathbbN$Proof of inequality involving multiplicative function?Bound for the sum of the divisors of a numberProve that $sum_n, d geq 1 = 2^omega(n)$Formula for unique distribution of colored balls into boxesUpper bound for the divisor counting function?How to invert an arithmetic function where Möbius inversion may not apply?










5












$begingroup$


I have the following exercise.




Show that for all natural numbers $n$, the following equality holds
$$sum_dmu(d)d(d)=(-1)^omega(n)$$
Here, $mu$ is the Möbius function, $d$ counts the number of divisors of $n$, and $omega$ counts the number of distinct prime divisors of $n$.




I tried looking at a number like $n=10$ just to see what it looks like expanded. So since the divisors of $n$ are $1,2,5,$ and $10$, I can show that
$$sum_10mu(d)d(d)=(-1)^omega(10)=mu(1)d(1)+mu(2)d(2)+mu(5)d(5)+mu(10)d(10)=1$$
This gives me
$$1cdot 1+-1cdot 2+-1cdot 2 +1cdot 4 $$
I'm thinking somehow since both $mu$ and $d$ are multiplicative that we can rewrite though as
$$mu(1)d(1)+mu(2)d(2)+mu(5)d(5)+mu(2)d(2)mu(5)d(5)$$
$$=mu(1)d(1)+mu(5)d(5)+mu(2)d(2)+mu(2)d(2)mu(5)d(5)$$
$$=mu(1)d(1)+mu(5)d(5)+mu(2)d(2)(1+mu(5)d(5))$$
$$=mu(1)d(1)+mu(5)d(5)+mu(2)d(2)(mu(1)d(1)+mu(5)d(5))$$
$$=(1+mu(2)d(2))(1+mu(5)d(5))$$
$$=sum_2mu(d)d(d)sum_dmu(d)d(d)$$
So this original summation function is multiplicative. But this isn't helping me see the how to move forward. I know that the $mu$ function is defined as $mu(n)=(-1)^omega(n)$ if $n$ is square free and $0$ if divisible by a square, so I think this plays a role somehow, but again, I'm feeling lost.



EDIT: Would looking at $n=prod_i=1^kp_i^alpha_i$ be a more useful approach to the problem? Knowing the larger summed function is multiplicative means I can focus my approach on the $p_i^alpha_i$...










share|cite|improve this question











$endgroup$











  • $begingroup$
    The book I am working out of is Niven's Introduction to the Theory of Numbers and they use $d$, so I've just gotten in the habit of using it. I know it's confusing. My professor did reveal that $tau$ was also used, but I guess I'm just used to the idea of $d$=divisor function.
    $endgroup$
    – Lalaloopsy
    Jul 19 '14 at 18:32






  • 5




    $begingroup$
    $d(d)$ is such a confusing notation...
    $endgroup$
    – CuriousGuest
    Jul 20 '14 at 6:54















5












$begingroup$


I have the following exercise.




Show that for all natural numbers $n$, the following equality holds
$$sum_dmu(d)d(d)=(-1)^omega(n)$$
Here, $mu$ is the Möbius function, $d$ counts the number of divisors of $n$, and $omega$ counts the number of distinct prime divisors of $n$.




I tried looking at a number like $n=10$ just to see what it looks like expanded. So since the divisors of $n$ are $1,2,5,$ and $10$, I can show that
$$sum_10mu(d)d(d)=(-1)^omega(10)=mu(1)d(1)+mu(2)d(2)+mu(5)d(5)+mu(10)d(10)=1$$
This gives me
$$1cdot 1+-1cdot 2+-1cdot 2 +1cdot 4 $$
I'm thinking somehow since both $mu$ and $d$ are multiplicative that we can rewrite though as
$$mu(1)d(1)+mu(2)d(2)+mu(5)d(5)+mu(2)d(2)mu(5)d(5)$$
$$=mu(1)d(1)+mu(5)d(5)+mu(2)d(2)+mu(2)d(2)mu(5)d(5)$$
$$=mu(1)d(1)+mu(5)d(5)+mu(2)d(2)(1+mu(5)d(5))$$
$$=mu(1)d(1)+mu(5)d(5)+mu(2)d(2)(mu(1)d(1)+mu(5)d(5))$$
$$=(1+mu(2)d(2))(1+mu(5)d(5))$$
$$=sum_2mu(d)d(d)sum_dmu(d)d(d)$$
So this original summation function is multiplicative. But this isn't helping me see the how to move forward. I know that the $mu$ function is defined as $mu(n)=(-1)^omega(n)$ if $n$ is square free and $0$ if divisible by a square, so I think this plays a role somehow, but again, I'm feeling lost.



EDIT: Would looking at $n=prod_i=1^kp_i^alpha_i$ be a more useful approach to the problem? Knowing the larger summed function is multiplicative means I can focus my approach on the $p_i^alpha_i$...










share|cite|improve this question











$endgroup$











  • $begingroup$
    The book I am working out of is Niven's Introduction to the Theory of Numbers and they use $d$, so I've just gotten in the habit of using it. I know it's confusing. My professor did reveal that $tau$ was also used, but I guess I'm just used to the idea of $d$=divisor function.
    $endgroup$
    – Lalaloopsy
    Jul 19 '14 at 18:32






  • 5




    $begingroup$
    $d(d)$ is such a confusing notation...
    $endgroup$
    – CuriousGuest
    Jul 20 '14 at 6:54













5












5








5


2



$begingroup$


I have the following exercise.




Show that for all natural numbers $n$, the following equality holds
$$sum_dmu(d)d(d)=(-1)^omega(n)$$
Here, $mu$ is the Möbius function, $d$ counts the number of divisors of $n$, and $omega$ counts the number of distinct prime divisors of $n$.




I tried looking at a number like $n=10$ just to see what it looks like expanded. So since the divisors of $n$ are $1,2,5,$ and $10$, I can show that
$$sum_10mu(d)d(d)=(-1)^omega(10)=mu(1)d(1)+mu(2)d(2)+mu(5)d(5)+mu(10)d(10)=1$$
This gives me
$$1cdot 1+-1cdot 2+-1cdot 2 +1cdot 4 $$
I'm thinking somehow since both $mu$ and $d$ are multiplicative that we can rewrite though as
$$mu(1)d(1)+mu(2)d(2)+mu(5)d(5)+mu(2)d(2)mu(5)d(5)$$
$$=mu(1)d(1)+mu(5)d(5)+mu(2)d(2)+mu(2)d(2)mu(5)d(5)$$
$$=mu(1)d(1)+mu(5)d(5)+mu(2)d(2)(1+mu(5)d(5))$$
$$=mu(1)d(1)+mu(5)d(5)+mu(2)d(2)(mu(1)d(1)+mu(5)d(5))$$
$$=(1+mu(2)d(2))(1+mu(5)d(5))$$
$$=sum_2mu(d)d(d)sum_dmu(d)d(d)$$
So this original summation function is multiplicative. But this isn't helping me see the how to move forward. I know that the $mu$ function is defined as $mu(n)=(-1)^omega(n)$ if $n$ is square free and $0$ if divisible by a square, so I think this plays a role somehow, but again, I'm feeling lost.



EDIT: Would looking at $n=prod_i=1^kp_i^alpha_i$ be a more useful approach to the problem? Knowing the larger summed function is multiplicative means I can focus my approach on the $p_i^alpha_i$...










share|cite|improve this question











$endgroup$




I have the following exercise.




Show that for all natural numbers $n$, the following equality holds
$$sum_dmu(d)d(d)=(-1)^omega(n)$$
Here, $mu$ is the Möbius function, $d$ counts the number of divisors of $n$, and $omega$ counts the number of distinct prime divisors of $n$.




I tried looking at a number like $n=10$ just to see what it looks like expanded. So since the divisors of $n$ are $1,2,5,$ and $10$, I can show that
$$sum_10mu(d)d(d)=(-1)^omega(10)=mu(1)d(1)+mu(2)d(2)+mu(5)d(5)+mu(10)d(10)=1$$
This gives me
$$1cdot 1+-1cdot 2+-1cdot 2 +1cdot 4 $$
I'm thinking somehow since both $mu$ and $d$ are multiplicative that we can rewrite though as
$$mu(1)d(1)+mu(2)d(2)+mu(5)d(5)+mu(2)d(2)mu(5)d(5)$$
$$=mu(1)d(1)+mu(5)d(5)+mu(2)d(2)+mu(2)d(2)mu(5)d(5)$$
$$=mu(1)d(1)+mu(5)d(5)+mu(2)d(2)(1+mu(5)d(5))$$
$$=mu(1)d(1)+mu(5)d(5)+mu(2)d(2)(mu(1)d(1)+mu(5)d(5))$$
$$=(1+mu(2)d(2))(1+mu(5)d(5))$$
$$=sum_2mu(d)d(d)sum_dmu(d)d(d)$$
So this original summation function is multiplicative. But this isn't helping me see the how to move forward. I know that the $mu$ function is defined as $mu(n)=(-1)^omega(n)$ if $n$ is square free and $0$ if divisible by a square, so I think this plays a role somehow, but again, I'm feeling lost.



EDIT: Would looking at $n=prod_i=1^kp_i^alpha_i$ be a more useful approach to the problem? Knowing the larger summed function is multiplicative means I can focus my approach on the $p_i^alpha_i$...







number-theory multiplicative-function divisor-counting-function mobius-inversion dirichlet-convolution






share|cite|improve this question















share|cite|improve this question













share|cite|improve this question




share|cite|improve this question








edited Mar 27 at 8:22









Eric Wofsey

193k14221352




193k14221352










asked Jul 19 '14 at 18:19









LalaloopsyLalaloopsy

89111221




89111221











  • $begingroup$
    The book I am working out of is Niven's Introduction to the Theory of Numbers and they use $d$, so I've just gotten in the habit of using it. I know it's confusing. My professor did reveal that $tau$ was also used, but I guess I'm just used to the idea of $d$=divisor function.
    $endgroup$
    – Lalaloopsy
    Jul 19 '14 at 18:32






  • 5




    $begingroup$
    $d(d)$ is such a confusing notation...
    $endgroup$
    – CuriousGuest
    Jul 20 '14 at 6:54
















  • $begingroup$
    The book I am working out of is Niven's Introduction to the Theory of Numbers and they use $d$, so I've just gotten in the habit of using it. I know it's confusing. My professor did reveal that $tau$ was also used, but I guess I'm just used to the idea of $d$=divisor function.
    $endgroup$
    – Lalaloopsy
    Jul 19 '14 at 18:32






  • 5




    $begingroup$
    $d(d)$ is such a confusing notation...
    $endgroup$
    – CuriousGuest
    Jul 20 '14 at 6:54















$begingroup$
The book I am working out of is Niven's Introduction to the Theory of Numbers and they use $d$, so I've just gotten in the habit of using it. I know it's confusing. My professor did reveal that $tau$ was also used, but I guess I'm just used to the idea of $d$=divisor function.
$endgroup$
– Lalaloopsy
Jul 19 '14 at 18:32




$begingroup$
The book I am working out of is Niven's Introduction to the Theory of Numbers and they use $d$, so I've just gotten in the habit of using it. I know it's confusing. My professor did reveal that $tau$ was also used, but I guess I'm just used to the idea of $d$=divisor function.
$endgroup$
– Lalaloopsy
Jul 19 '14 at 18:32




5




5




$begingroup$
$d(d)$ is such a confusing notation...
$endgroup$
– CuriousGuest
Jul 20 '14 at 6:54




$begingroup$
$d(d)$ is such a confusing notation...
$endgroup$
– CuriousGuest
Jul 20 '14 at 6:54










2 Answers
2






active

oldest

votes


















7












$begingroup$

Look at the sum for a prime power $p^k$. The divisors of $p^k$ are $1,p,p^2,...,p^k-1$. All of them contain a square except $1$ and $p$. That means $mu(p^a)=0$. So the sum is
$$mu(1)d(1)+mu(p)d(p)\=1times 1+(-1)times 2=1-2=-1$$
So each different prime, or its power, contributes a factor (-1).






share|cite|improve this answer









$endgroup$








  • 1




    $begingroup$
    In particular, since $mu(n)d(n)$ is a multiplicative function, so is $sum_n'mid n mu(n')d(n')$...
    $endgroup$
    – Thomas Andrews
    Jul 19 '14 at 18:32


















5












$begingroup$

Another, Combinatorial, way would be like $$sum _d mu (d) d(d)=sum _i=0^w(n)sum _p_1<p_2 ldots < p_imu (p_1 dots p_i)d (p_1 dots p_i)=sum _i=0^w(n)binomw(n)i(-1)^i2^i=(1-2)^w(n)$$ where the $p_j$ are primes in the descomposition of $n$.






share|cite|improve this answer









$endgroup$













    Your Answer








    StackExchange.ready(function()
    var channelOptions =
    tags: "".split(" "),
    id: "69"
    ;
    initTagRenderer("".split(" "), "".split(" "), channelOptions);

    StackExchange.using("externalEditor", function()
    // Have to fire editor after snippets, if snippets enabled
    if (StackExchange.settings.snippets.snippetsEnabled)
    StackExchange.using("snippets", function()
    createEditor();
    );

    else
    createEditor();

    );

    function createEditor()
    StackExchange.prepareEditor(
    heartbeatType: 'answer',
    autoActivateHeartbeat: false,
    convertImagesToLinks: true,
    noModals: true,
    showLowRepImageUploadWarning: true,
    reputationToPostImages: 10,
    bindNavPrevention: true,
    postfix: "",
    imageUploader:
    brandingHtml: "Powered by u003ca class="icon-imgur-white" href="https://imgur.com/"u003eu003c/au003e",
    contentPolicyHtml: "User contributions licensed under u003ca href="https://creativecommons.org/licenses/by-sa/3.0/"u003ecc by-sa 3.0 with attribution requiredu003c/au003e u003ca href="https://stackoverflow.com/legal/content-policy"u003e(content policy)u003c/au003e",
    allowUrls: true
    ,
    noCode: true, onDemand: true,
    discardSelector: ".discard-answer"
    ,immediatelyShowMarkdownHelp:true
    );



    );













    draft saved

    draft discarded


















    StackExchange.ready(
    function ()
    StackExchange.openid.initPostLogin('.new-post-login', 'https%3a%2f%2fmath.stackexchange.com%2fquestions%2f871933%2fprove-sum-k-mid-n-mukdk-1-omegan%23new-answer', 'question_page');

    );

    Post as a guest















    Required, but never shown

























    2 Answers
    2






    active

    oldest

    votes








    2 Answers
    2






    active

    oldest

    votes









    active

    oldest

    votes






    active

    oldest

    votes









    7












    $begingroup$

    Look at the sum for a prime power $p^k$. The divisors of $p^k$ are $1,p,p^2,...,p^k-1$. All of them contain a square except $1$ and $p$. That means $mu(p^a)=0$. So the sum is
    $$mu(1)d(1)+mu(p)d(p)\=1times 1+(-1)times 2=1-2=-1$$
    So each different prime, or its power, contributes a factor (-1).






    share|cite|improve this answer









    $endgroup$








    • 1




      $begingroup$
      In particular, since $mu(n)d(n)$ is a multiplicative function, so is $sum_n'mid n mu(n')d(n')$...
      $endgroup$
      – Thomas Andrews
      Jul 19 '14 at 18:32















    7












    $begingroup$

    Look at the sum for a prime power $p^k$. The divisors of $p^k$ are $1,p,p^2,...,p^k-1$. All of them contain a square except $1$ and $p$. That means $mu(p^a)=0$. So the sum is
    $$mu(1)d(1)+mu(p)d(p)\=1times 1+(-1)times 2=1-2=-1$$
    So each different prime, or its power, contributes a factor (-1).






    share|cite|improve this answer









    $endgroup$








    • 1




      $begingroup$
      In particular, since $mu(n)d(n)$ is a multiplicative function, so is $sum_n'mid n mu(n')d(n')$...
      $endgroup$
      – Thomas Andrews
      Jul 19 '14 at 18:32













    7












    7








    7





    $begingroup$

    Look at the sum for a prime power $p^k$. The divisors of $p^k$ are $1,p,p^2,...,p^k-1$. All of them contain a square except $1$ and $p$. That means $mu(p^a)=0$. So the sum is
    $$mu(1)d(1)+mu(p)d(p)\=1times 1+(-1)times 2=1-2=-1$$
    So each different prime, or its power, contributes a factor (-1).






    share|cite|improve this answer









    $endgroup$



    Look at the sum for a prime power $p^k$. The divisors of $p^k$ are $1,p,p^2,...,p^k-1$. All of them contain a square except $1$ and $p$. That means $mu(p^a)=0$. So the sum is
    $$mu(1)d(1)+mu(p)d(p)\=1times 1+(-1)times 2=1-2=-1$$
    So each different prime, or its power, contributes a factor (-1).







    share|cite|improve this answer












    share|cite|improve this answer



    share|cite|improve this answer










    answered Jul 19 '14 at 18:26









    Empy2Empy2

    33.7k12562




    33.7k12562







    • 1




      $begingroup$
      In particular, since $mu(n)d(n)$ is a multiplicative function, so is $sum_n'mid n mu(n')d(n')$...
      $endgroup$
      – Thomas Andrews
      Jul 19 '14 at 18:32












    • 1




      $begingroup$
      In particular, since $mu(n)d(n)$ is a multiplicative function, so is $sum_n'mid n mu(n')d(n')$...
      $endgroup$
      – Thomas Andrews
      Jul 19 '14 at 18:32







    1




    1




    $begingroup$
    In particular, since $mu(n)d(n)$ is a multiplicative function, so is $sum_n'mid n mu(n')d(n')$...
    $endgroup$
    – Thomas Andrews
    Jul 19 '14 at 18:32




    $begingroup$
    In particular, since $mu(n)d(n)$ is a multiplicative function, so is $sum_n'mid n mu(n')d(n')$...
    $endgroup$
    – Thomas Andrews
    Jul 19 '14 at 18:32











    5












    $begingroup$

    Another, Combinatorial, way would be like $$sum _d mu (d) d(d)=sum _i=0^w(n)sum _p_1<p_2 ldots < p_imu (p_1 dots p_i)d (p_1 dots p_i)=sum _i=0^w(n)binomw(n)i(-1)^i2^i=(1-2)^w(n)$$ where the $p_j$ are primes in the descomposition of $n$.






    share|cite|improve this answer









    $endgroup$

















      5












      $begingroup$

      Another, Combinatorial, way would be like $$sum _d mu (d) d(d)=sum _i=0^w(n)sum _p_1<p_2 ldots < p_imu (p_1 dots p_i)d (p_1 dots p_i)=sum _i=0^w(n)binomw(n)i(-1)^i2^i=(1-2)^w(n)$$ where the $p_j$ are primes in the descomposition of $n$.






      share|cite|improve this answer









      $endgroup$















        5












        5








        5





        $begingroup$

        Another, Combinatorial, way would be like $$sum _d mu (d) d(d)=sum _i=0^w(n)sum _p_1<p_2 ldots < p_imu (p_1 dots p_i)d (p_1 dots p_i)=sum _i=0^w(n)binomw(n)i(-1)^i2^i=(1-2)^w(n)$$ where the $p_j$ are primes in the descomposition of $n$.






        share|cite|improve this answer









        $endgroup$



        Another, Combinatorial, way would be like $$sum _d mu (d) d(d)=sum _i=0^w(n)sum _p_1<p_2 ldots < p_imu (p_1 dots p_i)d (p_1 dots p_i)=sum _i=0^w(n)binomw(n)i(-1)^i2^i=(1-2)^w(n)$$ where the $p_j$ are primes in the descomposition of $n$.







        share|cite|improve this answer












        share|cite|improve this answer



        share|cite|improve this answer










        answered Jul 21 '14 at 2:23









        PhicarPhicar

        2,8651915




        2,8651915



























            draft saved

            draft discarded
















































            Thanks for contributing an answer to Mathematics Stack Exchange!


            • Please be sure to answer the question. Provide details and share your research!

            But avoid


            • Asking for help, clarification, or responding to other answers.

            • Making statements based on opinion; back them up with references or personal experience.

            Use MathJax to format equations. MathJax reference.


            To learn more, see our tips on writing great answers.




            draft saved


            draft discarded














            StackExchange.ready(
            function ()
            StackExchange.openid.initPostLogin('.new-post-login', 'https%3a%2f%2fmath.stackexchange.com%2fquestions%2f871933%2fprove-sum-k-mid-n-mukdk-1-omegan%23new-answer', 'question_page');

            );

            Post as a guest















            Required, but never shown





















































            Required, but never shown














            Required, but never shown












            Required, but never shown







            Required, but never shown

































            Required, but never shown














            Required, but never shown












            Required, but never shown







            Required, but never shown







            Popular posts from this blog

            Lowndes Grove History Architecture References Navigation menu32°48′6″N 79°57′58″W / 32.80167°N 79.96611°W / 32.80167; -79.9661132°48′6″N 79°57′58″W / 32.80167°N 79.96611°W / 32.80167; -79.9661178002500"National Register Information System"Historic houses of South Carolina"Lowndes Grove""+32° 48' 6.00", −79° 57' 58.00""Lowndes Grove, Charleston County (260 St. Margaret St., Charleston)""Lowndes Grove"The Charleston ExpositionIt Happened in South Carolina"Lowndes Grove (House), Saint Margaret Street & Sixth Avenue, Charleston, Charleston County, SC(Photographs)"Plantations of the Carolina Low Countrye

            random experiment with two different functions on unit interval Announcing the arrival of Valued Associate #679: Cesar Manara Planned maintenance scheduled April 23, 2019 at 00:00UTC (8:00pm US/Eastern)Random variable and probability space notionsRandom Walk with EdgesFinding functions where the increase over a random interval is Poisson distributedNumber of days until dayCan an observed event in fact be of zero probability?Unit random processmodels of coins and uniform distributionHow to get the number of successes given $n$ trials , probability $P$ and a random variable $X$Absorbing Markov chain in a computer. Is “almost every” turned into always convergence in computer executions?Stopped random walk is not uniformly integrable

            How should I support this large drywall patch? Planned maintenance scheduled April 23, 2019 at 00:00UTC (8:00pm US/Eastern) Announcing the arrival of Valued Associate #679: Cesar Manara Unicorn Meta Zoo #1: Why another podcast?How do I cover large gaps in drywall?How do I keep drywall around a patch from crumbling?Can I glue a second layer of drywall?How to patch long strip on drywall?Large drywall patch: how to avoid bulging seams?Drywall Mesh Patch vs. Bulge? To remove or not to remove?How to fix this drywall job?Prep drywall before backsplashWhat's the best way to fix this horrible drywall patch job?Drywall patching using 3M Patch Plus Primer